a telemarketing company is conducting a study of new calling scripts. seventy-five employees will be randomly assigned to three new scripts using a table of random digits. the study leader will assign the subjects to the groups using a table of random digits. how many unique two-digit numbers within the range of 01 to 75 will the study designer need to select from the table? 3 25 50 75

Answers

Answer 1

Answer:

50

Step-by-step explanation:


Related Questions

PLSSSS HELPPPPP MEEE !!!!!

Answers

All the angles in a triangle add up to 180 degrees.

40 + x + (5x + 14) = 180

54 + 6x = 180

6x = 126

x = 21 degrees

Answer:

x=21

Step-by-step explanation:

a triangle adds up to 180 degree so to find x we will use the formula

(5x+14)+x+40=180

subtract 40 from each side so we have

(5x+14)+x=140

we have 2 terms with the same factor (x) so combine like terms

6x+14=140

subtract 14 from both sides

6x=126

divide both sides by 6

x=21

check your answer

5(21)+14

105+14=119

119+ 21+40=180

so x is equal to 21

PLSSS I NEED HELP
Which of the following is the graph ofY= -3/5x+2

Answers

Answer- C

Explanation - Since it’s a negative slope the line goes \ that way .

Answer:

It would be the last graph (c). It would look like the image below.

Explanation:

Since 3 is negative the answer would have a graph with a line going like this \.

Which of the following equations can be solved for x in one step by multiplying both sides of the equation by 2?

Answers

Answer:

x/2 = any number

Step-by-step explanation:

Find the length of a using
the Pythagorean Theorem.
8
a
6
Hint: a² + b² = c²
Round your answer to the nearest tenth.

Answers

Answer:5.3

Step-by-step explanation:

using the Pythagorean theorem 8²-6²=28    √28=5.3

5

Step-by-step explanation:

Pythagoras theorem states that

a²+b²=c²

therefore a²=b²-c²

a²=8²-6²

a=√8²-6²

=√64-36

=√28

=5.3 or approximately 5

euclid, pythagoras, ptolemy, and hypatia are playing a game where they all have to think of a number, and then cube that number 20 times. hypatia doesn't want to cube large numbers, so she chooses the number $1$. euclid thinks the same thing and also chooses the number $1$. however, pythagoras and ptolemy don't think ahead and pythagoras chooses $2$ and ptolemy chooses $-2$. after they finish cubing their numbers (pythagoras and ptolemy take a while), all four players write their final numbers on a piece of paper. what is the sum of the numbers they wrote on the piece of paper?

Answers

The sum of the numbers they wrote on the piece of paper is 2,305,843,009,213,693,954

What is Pythagoras's theorem?

The Pythagorean Theorem states that the squares on the hypotenuse (the side across from the right angle) of a right triangle or in standard algebraic notation, a² + b² = c², is equal to the squares on the legs.

What is the sum of numbers?

The outcome or conclusion we arrive at when we add two or more integers is known as the sum. Addends are the numbers that are added.

Here, we calculate the sum of the given numbers

= (1³)^²⁰  +  (1³)²⁰  +  (2³)²⁰  +  (-2³)²⁰

= 2,305,843,009,213,693,954

Hence, the sum of the numbers they wrote on the piece of paper is 2,305,843,009,213,693,954.

To learn more about the sum of numbers from the given link

https://brainly.com/question/28021413

#SPJ4

Mr karki divided a sum of Rs 1,80,000 between his son and daughter in the ratio of 4:5 find the sum obtained by eachof them.

Answers

Answer:

Rs 80,0000 and Rs 100,000

Step-by-step explanation:

sum the parts of the ratio, 4 + 5 = 9 parts

divide the amount by 9 to find the value of one part.

Rs 1,80,000 ÷ 9 = Rs 20,000 , then

4 parts = 4 × Rs 20,000 = Rs 80,000 ← amount son receives

5 parts = 5 × Rs 20,000 = Rs 100,000 ← amount daughter receives

The sum obtained by the son is Rs. 80,000 and the daughter is Rs. 1,00,000.

Mr. Karki divided Rs. 1,80,000 in a ratio of 4:5 between his son and daughter.

Let the common multiple be x

∴ The son obtained Rs. 4x        --------1

And daughter obtained Rs. 5x        --------2

As the total sum is Rs. 1,80,000

∴ 4x + 5x = 180000

9x = 180000

x = 180000/9

∴ x = Rs. 20000

Putting the value of x in 1 and 2 we get,

The sum obtained by son = 4x = 4*20000 = Rs. 80000

And the sum obtained by daughter = 5x = 5*20000 = Rs. 100000

Thus, the son obtained Rs. 80,000 while the daughter obtained Rs. 1,00,000 which is in the ratio of 4:5.

To learn more about ratios,

brainly.com/question/1504221

You and your friends go camping and put up a tent on camp grounds. the tent is shaped like an isosceles triangle. the sides of the tent are 5 feet each and the opening of the tent is 6 feet wide. how tall is the tent

Answers

Fortunately half the isosceles triangle forms a Pythagorean triple with 3, h, 5
h = 4 feet tall

Line K is represented by the equation y = -3/2x + 1. Line L is perpendicular to line K and passes through the point (6,2). Determine the equation is line L.

Answers

Answer:

y = (2/3)y - 2

Step-by-step explanation:

Linear equations of the form y=mx+b, describe the slope, m, and the y-intercept, b (the value of y when x=0).  

Line K

y = -(3/2)x + 1 tells us that it has a slope of -(3/2) and crosses the y axis (x=0) at y = 1.  

Perpendicular Lines

Perpendicular lines have a useful property in that they have a slope that is the "negative inverse" of the line they are perpendicular to.  If, for example, we were ever asked  what is the slope of a line perpendicular to Line L (yes, it happens), we could, with minimal effort, respond "It is the negative inverse of -(3/2)," which is a slope of (2/3).

Line L

We just accidently answered a key question when we found the negative inverse of the slope for Line K, (2/3).  That will be the slope of Line L that will make it perpendicular to Line K.

Line L will have the form y = (2/3)x + b

This line will be perpendicular to Line K, no matter the value of b.  But we want a perpendicular line that goes through point (6,2), so we need to find a value of b that will force it to go through that point.

The role b has in our equation is simply moving it up or down.  If b were 0, the line will go through the origin (0,0).  If b were 5, it would interect the y axis at 5 (0,5).  We could draw a graph and get a pretty good idea what b would have to be for the line to go throgh (6,2), but most prefer the easier route:  Use the (x,y) value in the above equation and solve for b.  Its easy, and works great:

   y = (2/3)x + b for (6,2)

  2 = (2/3)(6) + b

   b = 2 - (2/3)*(6)

   b = 2 - 4

    b = -2

The equation for Line L is thus:  y = (2/3)y - 2

See the attached graph.  Included are equations [marked Demo Line 1 and 2] with 2 random values of b (1 and 0), in order to:

demonstrate how b impacts the line, andsuggest try using DESMOS as a free online graphing tool.


Find the value of r so that the line passing through (-3, r) and (7,-6)
has a slope of 2 2/5
r =

Answers

The value of r is -10 for line with slope 2/5 .

What is slope?

A line's slope, often known as its gradient, is a numerical representation of the line's steepness and direction.

Main Body:

We know that the slope of line in two point form is

(y₂-y₁)/(x₂-x₁) = m where m = slope

given points are (-3,r) and (7,-6) m= 2/5

(-6-r)/(7+3)= 2/5

(-6-r)/10 =2/5

therefore value of r= -10

to learn more about slope click on the link below.

https://brainly.com/question/16949303

#SPJ13

this is my last question please help asap!!

Answers

The average rate of change of function [tex]g(x)=5(2)^{x}[/tex] from x = 3 to x = 4 is 4 times that from x = 1 to x = 2.

The correct option is (A).

What is the average rate of change of a function?

The average rate at which one quantity changes in relation to another's change is referred to as the average rate of change function.

Using function notation, we can define the Average Rate of Change of a function f from a to b as:

                                     [tex]rate(m) = \frac{f(b)-f(a)}{b-a}[/tex]

The given function is  [tex]g(x) = 5(2)^{x}[/tex],

Now calculating the average rate of change of function from x = 1 to x = 2.

                               [tex]m = \frac{g(b)-g(a)}{b-a}\\ m = \frac{g(2)-g(1)}{2-1}\\\\m=\frac{5(2)^{2}-5(2)^1}{2-1}\\ m=\frac{10}{1} \\m=10[/tex]

Now, calculate the average rate of change of function from x = 3 to x = 4.

                                 [tex]m = \frac{g(b)-g(a)}{b-a}\\ m = \frac{g(4)-g(3)}{4-3}\\\\m=\frac{5(2)^{4}-5(2)^3}{4-3}\\ m=\frac{40}{1} \\m=40[/tex]

The jump from m = 10 to m = 40 is "times 4".

So option (A) is correct.

Hence, The average rate of change of function [tex]g(x)=5(2)^{x}[/tex] from x = 3 to x = 4 is 4 times that from x = 1 to x = 2.

To learn more about the average rate of change of function, visit:

https://brainly.com/question/24313700

#SPJ1  

List all factors of the numerator and denominator. Use gcf method to simplify Creduce) each fraction. You must show work

Answers

The simplified form of the given fraction is, 2/3.

What is greatest common factor?

The largest gap between the two integers is considered to be the biggest common factor. Firstly, make a list of each number's prime factors to determine its largest common factor. The ages of 18 and 24 have one 2 and one 3. The GCF of 18 and 24 is obtained by multiplying them, therefore 2 * 3 = 6.

Consider, the given fraction [tex]\frac{48}{72}[/tex]

The divisors of 48 are, 1, 2, 3, 4, 6, 8, 12, 16, 24, 48.

The divisors of 72 are, 1, 2, 3, 4, 6, 8, 9, 12, 18, 24, 36, 72.

So, from above the greatest common factor of 48 and 72 is 24.

Therefore,

[tex]\frac{48}{72} = \frac{24(2)}{24(3)} = \frac{2}{3}[/tex]

Therefore, the simplified form of the given fraction is, 2/3.

To know more about the greatest common factor, click on the link

https://brainly.com/question/219464

#SPJ13

A) SAS
B) ASA
C) AAS
D) SSS
E) there is not enough information to guarantee congruent triangles

Answers

Pretty sure it’s option, A) SAS
As the parallel lines and angles give that information

if the product of five and the number is increased by seven, the result is the same as eight less than three times a number


Please helppppp

Answers

Answer:

The number is -7.5

Step-by-step explanation:

Creating an equation:

5x+7=3x-8

-> 5x = 3x-15

-> 2x=-15

-> x=-7.5

HALF SOLVED PLEASE HELP

Answers

its like the uh 26 yeah

PLEASE HELP FAST!!!! ILL GIVE BRAINLIEST!!!!

Which statement correctly compares the function shown on this graph with the function y = 4x - 8?

A. The function shown on the graph has a greater rate of change but
a lower y-intercept.

B. The function shown on the graph has a smaller rate of change but
a higher y-intercept.

C. The function shown on the graph has a greater rate of change and
a higher y-intercept.

D. The function shown on the graph has a smaller rate of change and
a lower y-intercept.

Answers

I think The answer to your question is c

What is 10 and 3/4 as a decimal

Answers

Answer:10.75

Step-by-step explanation:

3/4 as a decimal is 0.75. add that to the 10 gives you 10.75

Write the linear equation in the slope intercept form.

(-6,0); slope= 2/3

Answers

Answer:

Step-by-step explanation:

y=2/3x+4

a 39-inch by 104-inch piece of cardboard is used to make an open-top container by removing a square from each corner of the cardboard and folding up the flaps on each side. what size square should be cut from each corner to get a container with the maximum volume? enter the area of the square and do not include any units in your answer.

Answers

According to the solving the area of the square is 68.0625.

What's a square's area?

As is common knowledge, a square is a four-sided, two-dimensional figure. It is also referred to as a quadrilateral. The total quantity of unit squares forming a square is referred to as the square's area. In other words, it is described as the area that the square takes up.

According to the given data:

The box formed after cutting the square from each corner will have the dimensions as,

length = 104 - 2x, width = 39 - 2x, height = x.

∴ volume of the box = length × width × height

∴ v = (104 - 2x)(39 - 2x)(x) -----(i)

∴ v = (104 - 2x) (39x - 2[tex]x^{2}[/tex])

∴ v = 104(39x - 2[tex]x^{2}[/tex]) -2x(39x - 2[tex]x^{2}[/tex])

∴ v = 4056x - 208[tex]x^{2}[/tex] - 78[tex]x^{2}[/tex] + 4[tex]x^{3}[/tex]

∴ v = 4[tex]x^{3}[/tex] - 286[tex]x^{2}[/tex]  + 4056x

let f(x) =  4[tex]x^{3}[/tex] - 286[tex]x^{2}[/tex]  + 4056x  -----(ii)

To, find x for which f(x) is maximum,

⇒we should apply second derivative test ,

According to this test, first we should find critical points at which f'(x) = 0.

then if f''(x) < 0 for that critical point then f(x) is maximum at that critical point.

∴ let us consider, f'(x) = 0.

now, f(x) =  4[tex]x^{3}[/tex] - 286[tex]x^{2}[/tex]  + 4056x

⇒ f'(x) = 12[tex]x^{2}[/tex] - 572x + 4056.  -----(iii)

⇒ f'(x) = 4(3[tex]x^{2}[/tex] - 143x + 1014)

⇒ f'(x) = 0.

⇒  f'(x) = 4(3[tex]x^{2}[/tex] - 143x + 1014) = 0

⇒ x = (-b ± [tex]\sqrt{b^{2} - 4ac }[/tex])/2a    ; where a = 3, b = -143, c = 1014.

∴ x = (-(-143) ± [tex]\sqrt{(-143)^{2} -4(3)(1014)}[/tex])/2×3

∴ x = (143 ± [tex]\sqrt{8281}[/tex])/6.

∴ x = [tex]\frac{143 + 91}{6}[/tex]  , x = [tex]\frac{143 - 91}{6}[/tex]

⇒ x = 39, 8.25

the square of length 8.25 inch should be cut from each side to det contained with the maximum volume.

Area of the square is = [tex]x^{2}[/tex] = [tex]8.25^{2}[/tex]

∴ Area = 68.0625

know more about area of square visit this link:

https://brainly.com/question/1658516

#SPJ4

A research association reported that 3,173,000 gas grills were shipped by various manufacturers in the U.S. in 2000.Shipment increased by 7% per year from 2000-2016.Write a function that models the number of gas grills shipped over time.

Answers

If shipments increased by 7% (which is the same as 0.07) per year, then the shipment in a year can be found by multiplying the shipments in the previous year by 1.07.

Therefore, if [tex]x[/tex] represents the number of years since 2000 and [tex]f(x)[/tex] represents the number of shipments, then [tex]f(x)=3173000(1.07)^x[/tex].

in a particular chi-square goodness-of-fit test, there are eight categories and 825 observations. use the 0.10 significance level. a. how many degrees of freedom are there?

Answers

By probability , 7 is degrees of freedom are there .

Describe probability using an example?

By dividing the number of preferable possibilities by the total number of potential outcomes, probability, which measures the likelihood that an event will occur, is obtained. The most basic illustration is a coin toss. There are only two outcomes that can occur when you flip a coin: either heads or tails.It is predicated on the likelihood that something will occur. The theory of probability primarily relies on the justification for probability. A coin is tossed, for instance, and the theoretical likelihood of getting a head is 1 in 2.

825 Observations and 8 categories .

Degree of freedom = k - 1

     df = 8 - 1

     df = 7

Learn more about probability

brainly.com/question/11234923

#SPJ4

HELP
(2, -1); m = 5/4


Answers

Answer:

part b

y + 1 = 5/4(x - 2)  

part c

y - 5 = 3(x - 0) or y - 5 = 3(x)

Step-by-step explanation:


Use the given vertices to graph quadrilateral TUVW and its image after a dilation centered at the origin with scale factor
k=2/3
T(9,-3), U(6,0), V(3,9), w(0,0)

Answers

Answer:We know that the dilation transformation of a figure either shrink or expand the original figure depending on the scale factor of the dilation.

As here we have scale factor=3>1.

Hence, the dilation  changes the original will expand the original figure.

Hence, each of the coordinate of a figure get multiplied by 3.

Hence, the co

ordinates of the image are given as:

i.e. A(0,0) → A'(0,0)

B(2,3) → B'(6,9)

C(1,-2) → C'(3,-6)

D(-3,-3) → D'(-9,-9)

Step-by-step explanation:

What is 12 = -4(-6x - 3)

Answers

Answer:

no solution

Step-by-step explanation:

12=-4(-6x-3)

12=24x+12

0=24x

Compute and expressed the result as a mixed fraction?

1) 2+ 3/4

Answers

Answer:

2 [tex]\frac{3}{4}[/tex]

Step-by-step explanation:

2 + [tex]\frac{3}{4}[/tex]

= 2 [tex]\frac{3}{4}[/tex] ← as a mixed number

If was asked to think of a number and multiply it by 2, could
write this algebraically as 2x.
Write the following algebraically, using x as your unknown.
"I think of a number, multiply it by 5 and subtract 3 from the result."

Answers

The algebraic expression of the mentioned statement will be 5x-3.

According to the question,

An unknown number is thought of, multiplied by 5, and then subtracted by 3 from the result.

Let the unknown number be taken as x.

If we have to write the statement mathematically by taking x as a variable, the algebraic expression will be,

x* (5), which is equal to 5x, (by taking the variable x and multiplying it with 5).

For the second step, the expression will be 5x -3, (by subtracting 3 from the previous result, that is, 5x).

Hence, algebraically the final statement will be 5x -3.

To learn more about Algebraic expressions:

https://brainly.com/question/19864285

Use polynomial long division to find the quote and the remainder when 2x^3-4x^2-x+2 is divided by X+2

Answers

The quotient of the polynomial is 4x - 9 and the remainder is 20

What are Quotient and remainder?

The resultant of the division is called the quotient while the Remainder is the number that is left after division.

In long division, the terms on top of the division symbol are called the quotient while that at the bottom is called the remainder.

When a division has no remainder it means that the polynomial is divisible by the divisor.

The body of the calculation is found in the attached file below.

In conclusion, the Quotient and remainder are 4x - 9 and 20 respectively.

Learn more about quotient and remainder: https;//brainly.com/question/18191517

#SPJ1

Give the numerical coefficient of the term.
xw

The numerical coefficient is _____.

Answers

Answer:

x

Step-by-step explanation:

The numerical coefficient is x.

cook-it rice cooker has a mean time before failure of 38 months with a standard deviation of 4 months, and the failure times are normally distributed. what should be the warranty period, in months, so that the manufacturer will not have more than 7% of the rice cookers returned? round your answer down to the nearest whole number.

Answers

In 40 months, so that the manufacturer will not have more than 7% of the rice cookers returned.

Define p- value.

A p-value calculates the likelihood of getting the outcomes that were observed, presuming that the null hypothesis is correct. The statistical significance of the difference that was found increases with decreasing p-value. Statistical significance is commonly defined as a p-value of 0.05 or less. The sample data, test type, and sampling distribution of the test statistic under the null hypothesis are used to determine the p-value (lower-tailed test, upper-tailed test, or two-sided test).

Given,

Cook-it rice cooker has a mean time before failure of 38 months with a standard deviation of 4 months, and the failure times are normally distributed.

The 7th percentile, or X when Z has a p-value of 0.08, or X when Z = 0.4720, should serve as the warranty period.

Z = X - Mean/ standard deviation

0.472 = x - 38/4

0.472 (4) - x - 38

1.888 = x - 38

x = 39.88

x = 40

In 40 months, so that the manufacturer will not have more than 7% of the rice cookers returned.

To learn more about p- value, visit:

https://brainly.com/question/24241687

#SPJ4

TR is the angle bisector angle ATS. The m angle RTS = (17x-5) and m angle ATS= (28x + 8). find X

Answers

Applying the definition of angle bisector, the value of x is: x = 3.

What is an Angle Bisector?

An angle bisector can be defined as a ray or line that divides an angle into two parts that are congruent or equal to each other in measure.

Since TR is the angle bisector that bisects angle ATS, it implies that angle ATS is divided into equal parts.

Given the following:

Measure of angle RTS = (17x - 5)°

Measure of angle ATS = (28x + 8)°

Therefore:

Measure of angle ATS = 2(measure of angle RTS) [definition of angle bisector]

Substitute

28x + 8 = 2(17x - 5)°

28x + 8 = 34x - 10

Combine like terms

28x - 34x = -8 - 10

-6x = -18

Divide both sides by -6

-6x/-6 = -18/-6

x = 3

Learn more about angle bisector on:

https://brainly.com/question/24334771

#SPJ1

someone please help!

Answers

L = 66 degrees
LK = 3.6
LM = 8.8
Other Questions
Question 151 ptsWhat are the common goals of both fiscal and monetarypolicy? Two forces are applied to a car in an effort to accelerate it, as shown below. The first force, F1 = 310 N, is applied at an angle = 32 to the forward dashed line. The second force, F2 = 464 N, is applied at an angle = 13 to the forward dashed line.(a) What is the resultant of these two forces? ______N at _____ to the _____(b) If the car has a mass of 2850 kg, what acceleration does it have? (Disregard friction.) _____m/s2 at ______ to the _____ What are the slope and vertical intercept of 5x+7y=10 what is one way the vision of primates is different from that of other animals? group of answer choices bjorn is trying to estimate a difference in average sugar content in two different types of donut: one that is fat free, and one that is full fat. in order to do this, he collects 60 donuts to test: 22 donuts are fat free, 38 that are full fat. he says this satisfies his assumptions. is he right? if not what's wrong? (select all that apply) How much more per week does a college grad earn compared to someone with some college?. What is a main reason you would use an object to help you understand a topic?(1 point) Responses It allows you to see something real. It allows you to see something real. It allows you to cite sources easily. It allows you to cite sources easily. It allows you to use multiple senses. It allows you to use multiple senses. It allows you to use bullet points. Readers Response #7Write a paragraph in response to each of the following three prompts:Why does Kingstons mother tell her the story of her aunt? What message does it send about women and womens roles? Why is the aunt a No Name Woman?What is the message of Chimamanda Adichies TED Talk The Danger of a Single Story? Compare Hong Kingstons story to Chimamanda Adichies TED Talk. How are they similar? What are they saying about stories, voice, and identity? Choose the warmer temperature. -29f or -39 suppose you purchased a stock a year ago. today, you receive a dividend of $18 and you sell the stock for $114. if your return was 13%, at what price did you buy the stock? $ . Why did W.E.B. Du Bois focus on the issue of voting rights? How was his opinion similar or different from the way the women's movement viewed voting rights? how did the national theater of the deaf (ntd) create a deaf-focused theatrical experience that differed from traditional theaters? Select "Correct" if the sentence is punctuated correctly and "Run-on" if it is not.Because light bulbs were on sale, I bought five packages.A) CorrectB) Run-on What is the solution to the system graphed below? PLS ANSWER WILL MARK BRAINLIST betina is trying to convince a buyer to purchase her company. the buyer wants to know what the company's overall financial condition was on december 31, 2018. what type of financial statement should betina provide? cash flow statement balance sheet statement of changes in equity income statement Read the poem "Crowned" by Amy Lowell and then answer the question that follows.You came to me bearing bright roses,Red like the wine of your heart;You twisted them into a garlandTo set me aside from the mart.Red roses to crown me your lover,And I walked aureoled and apart.Enslaved and encircled, I bore it,Proud token of my gift to you.The petals waned paler, and shriveled, And dropped; and the thorns started through.Bitter thorns proclaim me your lover,A diadem woven with rue.If the roses symbolize the love that the speaker's partner has for her, what do the thorns symbolize?A) the ongoing strength of the relationshipB) overcoming struggles in a relationshipC) the negative parts of the relationshipD) their love fading From 1809-1817, ___ served as the 4th president of the united states. He helped draft the constitution and some of the federalist papers. It is estimated that only around 200,000 Americans have central heterochromia (two different eye colors in the same eye). If the population of the United Stateswas 327,750,000 people at the time of this report, what percent of Americans had central heterochromia?0.0061%0.00061%0.61%0.061% Madison needs to order some new supplies for the restaurant where she works. The restaurant needs at least 474 spoons. There are currently 246 spoons. If each set on sale contains 6 spoons, use the drop-down menu below to write an inequality representing s, the number of sets of spoons Madison should buy. What spurred or inspired Haidri to write the poem Lottery?